Slippery Sum!

Calculus Level 4

r = 1 r ( r + 1 ) 3 r \displaystyle\sum _{ r=1 }^{ \infty }{ \frac { r\left( r+1 \right) }{ { 3 }^{ r } } }

If the summation above equals to m n \frac m n for coprime positive integers m , n m,n , what is the value of m + n m+n ?


The answer is 13.

This section requires Javascript.
You are seeing this because something didn't load right. We suggest you, (a) try refreshing the page, (b) enabling javascript if it is disabled on your browser and, finally, (c) loading the non-javascript version of this page . We're sorry about the hassle.

4 solutions

Discussions for this problem are now closed

Ahmed Arup Shihab
Feb 21, 2015

S = 1.2 3 + 2.3 3 2 + 3.4 3 3 + S= \frac{1.2}{3}+ \frac{2.3}{3^2}+ \frac{3.4}{3^3}+ \dots

Dividing both side by 3

S 3 = 1.2 3 2 + 2.3 3 3 + 3.4 3 4 + \frac{S}{3} = \frac{1.2}{3^2}+ \frac{2.3}{3^3}+ \frac{3.4}{3^4}+ \dots

Subtracting second equation from the first,

2 S 3 = 1.2 3 + 2.2 3 2 + 3.2 3 3 + 4.2 3 4 + \frac{2S}{3} = \frac{1.2}{3}+ \frac{2.2}{3^2}+ \frac{3.2}{3^3}+\frac{4.2}{3^4}+ \dots

S 3 = 1 3 + 2 3 2 + 3 3 3 + 4 3 4 + ( 1 ) \Rightarrow \frac{S}{3}= \frac{1}{3}+ \frac{2}{3^2}+ \frac{3}{3^3}+\frac{4}{3^4}+ \dots \cdots(1)

Again Dividing both side by 3 and subtracting from (1),

2 S 9 = 1 3 + 1 3 2 + 1 3 3 + 1 3 4 + \frac{2S}{9}=\frac{1}{3}+ \frac{1}{3^2}+ \frac{1}{3^3}+\frac{1}{3^4}+ \dots

S = 9 2 × 1 3 1 1 3 = 9 4 \Rightarrow S= \frac{9}{2} \times \frac{\frac{1}{3}}{1-\frac{1}{3}} = \frac{9}{4}

So answer is 9 + 4 = 13 9+4=\fbox{13}

i understood ur explanation quite well....though i didn't understand why division by 3 and not by anything else and why subtraction and not any other operation.. sorry to ask too stupid qn ..would be happy to see ur reply

Shehanaaz Sk - 6 years, 3 months ago

Just to offer a different approach to Atul's solution, note first that

r = 1 x r + 1 = x 2 1 x \displaystyle\sum_{r=1}^{\infty} x^{r+1} = \dfrac{x^{2}}{1 - x} for x < 1. |x| \lt 1.

Differentiating both sides with respect to x x , (the LHS term-by-term), and then simplifying, we have that

r = 1 ( r + 1 ) x r = 2 x x 2 ( 1 x ) 2 . \displaystyle\sum_{r=1}^{\infty} (r + 1)x^{r} = \dfrac{2x - x^{2}}{(1 - x)^{2}}.

Differentiating again and simplifying, we find that

r = 1 r ( r + 1 ) x r = 2 x ( 1 x ) 3 . \displaystyle\sum_{r=1}^{\infty} r(r + 1)x^{r} = \dfrac{2x}{(1 - x)^{3}}.

Plugging in x = 1 3 x = \dfrac{1}{3} we find that S = 2 3 ( 2 3 ) 3 = 9 4 , S = \dfrac{\frac{2}{3}}{(\frac{2}{3})^{3}} = \dfrac{9}{4},

and thus m + n = 9 + 4 = 13 . m + n = 9 + 4 = \boxed{13}.

Nice one .............. :)

Ahmed Arup Shihab - 6 years, 3 months ago

Thanks. I like your clever non-calculus approach as well. There is a good diversity of solutions here for people to analyze. :)

Brian Charlesworth - 6 years, 3 months ago

Wow, this is quite a neat solution. Is there a website that I could learn more about this from?

Trevor Arashiro - 6 years, 3 months ago

This video is fairly helpful, and he has several more videos that might be useful as well.

Brian Charlesworth - 6 years, 3 months ago

the expansion of ( 1 x ) n { \left( 1-x \right) }^{ -n } is as follows

( 1 x ) n { (1-x) }^{ -n } = 1 + ( n 1 ) x + ( n + 1 2 ) x 2 + ( n + 2 3 ) x 3 + . . . . . . =\quad 1\quad +\quad \left( \begin{matrix} n \\ 1 \end{matrix} \right) x\quad +\quad \left( \begin{matrix} n+1 \\ 2 \end{matrix} \right) { x }^{ 2 }\quad +\quad \left( \begin{matrix} n+2 \\ 3 \end{matrix} \right) { x }^{ 3 }\quad +...... = 1 + ( n n 1 ) x + ( n + 1 n 1 ) x 2 + ( n + 2 n 1 ) + . . . . =\quad 1\quad +\quad \left( \begin{matrix} n \\ n-1 \end{matrix} \right) x\quad +\left( \begin{matrix} n+1 \\ n-1 \end{matrix} \right) { x }^{ 2 }\quad +\quad \left( \begin{matrix} n+2 \\ n-1 \end{matrix} \right) \quad +....

where ( n r ) \left( \begin{matrix} n \\ r \end{matrix} \right) = n ! ( n r ) ! r ! \frac { n! }{ (n-r)!r! } .

Putting n=3 we get

( 1 x ) 3 { \left( 1-x \right) }^{ -3 } = 1 + ( 3 2 ) x + ( 4 2 ) x 2 + ( 5 2 ) x 3 + . . . . =\quad 1\quad +\quad \left( \begin{matrix} 3 \\ 2 \end{matrix} \right) x\quad +\left( \begin{matrix} 4 \\ 2 \end{matrix} \right) { x }^{ 2 }\quad +\quad \left( \begin{matrix} 5 \\ 2 \end{matrix} \right) { x }^{ 3 }\quad +....

Now multiplying both sides by 2x and substituting x=1/3 we get required sum.

2 × 1 / 3 ( 1 1 / 3 ) 3 = 1.2 3 + 2.3 3 2 + 3.4 3 3 + . . . { 2\times 1/3\left( 1-1/3 \right) }^{ -3 }=\quad \frac { 1.2 }{ 3 } +\frac { 2.3 }{ { 3 }^{ 2 } } +\frac { 3.4 }{ { 3 }^{ 3 } } +...

= 9 4 =\frac { 9 }{ 4 }

Nice problem! :)

Pranjal Jain - 6 years, 3 months ago
Jakub Kocák
Feb 22, 2015

I would like to write more heuristic and mathematically correct solution

First of all, this sum looks like good candidate for power series in form

S ( x ) = n = 0 + n ( n + 1 ) x n , S(x) = \displaystyle \sum_{n=0}^{+\infty} n (n+1) x^n \, ,

for x = 1 3 x = \frac{1}{3} (there in no difference if we omit term for n = 0 n= 0 or we don't). Here is important observation. Only power series with similar form we can directly calculate is geometric series

f ( x ) = n = 0 + x n = 1 1 x , f(x) = \displaystyle \sum_{n=0}^{+\infty} x^n = \frac{1}{1-x} \, ,

which converges for x < 1 |x| < 1 (this condition is fulfilled for x = 1 3 x = \frac{1}{3} ). Using derivative of power series, we could obtain the needed coefficient n ( n + 1 ) n (n+1) .

For power series we know, that within radius of convergence of power series we can derive power series term by term.

When

F ( x ) = n = 0 + a n x n , F(x) = \displaystyle \sum_{n=0}^{+\infty} a_n x^n \, ,

for x ( R , + R ) x \in (-R,+R ) , then

d F d x ( x ) = n = 0 + d d x ( a n x n ) = n = 0 + a n n x n 1 , \frac{\mathrm{d} F }{\mathrm{d} x } (x) = \displaystyle \sum_{n=0}^{+\infty} \frac{\mathrm{d} }{\mathrm{d} x } (a_n x^n) = \displaystyle \sum_{n=0}^{+\infty} a_n n x^{n-1} \, ,

for x ( R , + R ) x \in (-R,+R ) .

Now we do second derivative of f ( x ) f(x) :

f ( x ) = n = 0 + n x n 1 = n = 1 + n x n 1 = 1 ( 1 x ) 2 , f'(x) = \displaystyle \sum_{n=0}^{+\infty} n x^{n-1} = \displaystyle \sum_{n=1}^{+\infty} n x^{n-1} = \frac{1}{(1-x)^2} \, ,

f ( x ) = n = 1 + n ( n 1 ) x n 2 = n = 2 + n ( n 1 ) x n 2 = 2 ( 1 x ) 3 . f''(x) = \displaystyle \sum_{n=1}^{+\infty} n (n-1) x^{n-2} = \displaystyle \sum_{n=2}^{+\infty} n (n-1) x^{n-2} = \frac{2}{(1-x)^3} \, .

Now we denote m = n 1 m = n -1 to obtain

f ( x ) = m = 1 + ( m + 1 ) m x m 1 = 2 ( 1 x ) 3 . f''(x) = \displaystyle \sum_{m=1}^{+\infty} (m+1) m x^{m-1} = \frac{2}{(1-x)^3} \, .

Especially for x = 1 3 x = \frac{1}{3}

m = 1 + ( m + 1 ) m ( 1 3 ) m 1 = 27 4 . \displaystyle \sum_{m=1}^{+\infty} (m+1) m \left( \frac{1}{3} \right)^{m-1} = \frac{27}{4} \, .

Dividing equation with 3 we obtain answer

m = 1 + m ( m + 1 ) 3 m 1 1 3 = m = 1 + m ( m + 1 ) 3 m = 9 4 , \displaystyle \sum_{m=1}^{+\infty} \frac{m (m+1) }{3^{m-1}} \frac{1}{3} = \displaystyle \sum_{m=1}^{+\infty} \frac{m (m+1) }{3^m} = \frac{9}{4} \, ,

and thus answer is 9 + 4 = 13 9 + 4 = 13 .

0 pending reports

×

Problem Loading...

Note Loading...

Set Loading...